Not absolutely sure this is a bug but can't figure out why the Holt linear trend equations produces different slope values to those given by the components() view of an ETS model. I have posted a reproducible example on Cross Validated
https://stats.stackexchange.com/questions/630437/i-cannot-replicate-the-slope-for-an-ets-model-with-alpha-0-5-and-beta-0-5-usi
I am not skilled enough to look through the code, but hope that you can confirm that it should be possible to replicate the level and trend using the Holt equations.
Philip